5
$\begingroup$

I have a fundamental question regarding conditional probability. Lets say I have $n$ independent random variables $X_1, X_2, \ldots, X_n$. Another random variable, $W$ is conditioned on the conjunction of these random variables: $P(W | X_1, X_2, X_3, \ldots, X_n)$. Given that $X_1, \ldots, X_n$ are independent, is it possible to write

$P(W | X_1, X_2, X_3, \ldots, X_n) = P(W | X_1) \cdot P(W | X_2) \cdot \cdots \cdot P(W | X_n).$

Or is the only way to rewrite is the Bayes theorem?

  • 0
    You can use $\TeX$ on this site by enclosing formulas in dollar signs; single dollar signs for inline formulas and double dollar signs for displayed equations. You can see the source code for any math formatting you see on this site by right-clicking on it and selecting "Show Math As:TeX Commands". [Here](http://meta.math.stackexchange.com/questions/5020/mathjax-basic-tutorial-and-quick-reference)'s a basic tutorial and quick reference. There's an "edit" link under the question.2012-09-27

2 Answers 2

3

That expression is usually false for $n \gt 1$.

For example if $W$ is independent of all the $X_i$ and if $P(W)=p$ then the left hand side is $p$ while the right hand side is $p^n$, which is rather smaller.

  • 0
    @Marie: My counter-example was an example. The main point is that karthik $A$ is multiplying a lot of fractions together on the right hand side, and this will usually (though not always) be smaller than the left hand side2013-04-03
2

The answer is no, it is no possible in general. Here a counterexample:

Let the sample space be $\{(0,0),(0,1),(1,0),(1,1)\}$ and the events:

  • $A$: The first element is one, or $\{(1,0),(1,1)\}$.
  • $B$: The second element is one, or $\{(0,1),(1,1)\}$.

Then is easy to see that:

  • $P(A)=\frac{1}{2}$ (First element one)
  • $P(B)=\frac{1}{2}$ (Second element one)
  • $P(A,B)=\frac{1}{4}$ (Both ones)
  • $P(A)P(B)=\frac{1}{4}$

Hence, $A$ and $B$ are independents. Now, let:

$W$: Both ones, or $\{(1,1)\}$ (this is the same as A,B)

Then:

  • $P(W|A)=\frac{1}{2}$
  • $P(W|B)=\frac{1}{2}$

But

$P(W|A,B)=1$ Since W={A,B} (Both elements one). Also, $ P(W|A)P(W|B)=\frac{1}{4}$

so $P(W|A,B) \neq P(W|A)P(W|B)$